HS 328 Investments

Pataasin ang iyong marka sa homework at exams ngayon gamit ang Quizwiz!

An investor is close to retirement and is worried about the possibility of his stock portfolio suddenly losing value. He has heard that certain options strategies can limit his downside risk, but he is hesitant to pay premiums each month. Which options strategy, if structured appropriately, could limit his downside risk without costing him anything out of pocket? A) A collar B) A protective put C) A covered call D) A long straddle

The correct answer is (A). A collar combines a protective put and a covered call. A zero-cost collar is created when the premiums received from the short call offset the premiums paid for the long put.

A hybrid mutual fund is one that A) includes a mix of asset classes, such as stocks and bonds. B) has properties of both open-end and closed-end funds. C) offers opportunities for both income and capital appreciation. D) includes a mix of domestic and foreign securities.

The correct answer is (A). A hybrid mutual fund includes a mix of asset classes, such as stocks and bonds.

A high-income investor owns a portfolio of bonds. Interest rates abruptly and sharply increased. Which of the following strategies would be most helpful in order to take advantage of the decrease in the value of this investor's fixed-income securities? A) The tax swap B) The spread swap C) The bullet strategy D) The barbell strategy

The correct answer is (A). As interest rates increase, the price of bonds decreases. This drop in value may create an opportunity to recognize the capital loss for tax purposes, which is the purpose of the tax swap.

Brandy is considering purchasing an 8-year bond that is selling for $700. What is the current yield for this bond if it has a 6% coupon, paid semiannually? A) 8.57% B) 9.32% C) 10.17% D) 11.92%

The correct answer is (A). Current yield = $60/$700 = 8.57%

A mutual fund's returns over the past 4 years were -4%, -15%, 13%, and 13%. What was its geometric mean return over these years? A) 1.03% B) 1.75% C) 2.08% D) 11.17%

The correct answer is (A). GMR = [(1 + -4%) × (1 + -15%) × (1 + 13%) × (1 + 13%)](1/4) − 1 GMR = [(0.96) × (0.85) × (1.13) × (1.13)](1/4) − 1 GMR = [1.0420](1/4) − 1 GMR = 1.0103 − 1 = 0.0103 = 1.03%

An investor, who purchases a large collection of securities with no effort spent on individual security evaluation, is engaging in which approach to the asset-selection decision? A) Indexing B) Benchmarking C) Active management D) Market timing

The correct answer is (A). Indexing is a form of passive investing in which the investor simply purchases an index of returns using mutual funds or exchange-traded funds.

An investment manager routinely uses technical analysis to outperform his benchmark portfolio. He also finds that fundamental analysis and a little bit of insider trading (which is illegal) provide improved returns. Based on this information, he is investing in a market that is A) inefficient. B) weak-form efficient. C) semi-strong form efficient. D) strong-form efficient.

The correct answer is (A). Technical analysis should not provide improved returns in any type of efficient market. This market is inefficient.

Which of the following is accurate regarding the capital market line (CML)? A) The portfolios on the new efficient frontier are some combination of the risk-free asset and the market portfolio. B) The CML is a line that begins at the risk-free rate of return and crosses the efficient frontier at the market portfolio. C) The CML represents the most efficient portfolios of individual stocks. D) The slope of the CML is an investor's indifference curve.

The correct answer is (A). The CML is a combination of the risk-free return and the market portfolio. Option (B) is incorrect, as the CML does not cross the efficient portfolio, rather it is tangent to it. Option (C) is incorrect as the CML consists of the market portfolio and the Rf. Option (D) is incorrect as the slope is the Sharpe Ratio.

Which securities law regulates the offering and sale of securities in the primary market and ensures more transparency in financial statements? A) The Securities Act of 1933 B) The Securities Exchange Act of 1934 C) The Investment Advisers Act of 1940 D) The Investment Company Act of 1940

The correct answer is (A). The Securities Act of 1933 requires disclosures of new securities in the primary market. The Securities Exchange Act of 1934 focuses on the trading of securities in the secondary market. The Investment Advisers Act of 1940 regulates investment advisers and requires registration with the SEC for firms or any individual advisers with assets under management exceeding $100 million. The Investment Company Act of 1940 forms the backbone of financial regulation and established the foundation for mutual funds and hedge funds.

Joe is a 24% federal income taxpayer. His capital-gains rate and qualified dividend rate is 15%. Joe bought 100 shares of ING for $12 per share 10 months ago. ING paid a dividend of $80 during Joe's holding period. Joe just sold all the stock for $20 per share. What are Joe's tax consequences for the year? A) $80 taxed at the LTCG rate, $800 taxed at the ordinary income rate B) $80 taxed at the ordinary income rate, $800 taxed at the LTCG rate C) $720 taxed at the ordinary income rate D) $880 taxed at the LTCG rate

The correct answer is (A). The dividend is a qualified dividend taxed at the LTCG rate. The gain is a short-term capital gain and is taxed at ordinary income tax rates. SP $2,000 PP ($1,200) Gain $800 (STCG)

Five years ago, a married couple purchased a home for $500,000 and has lived there since. They just sold the home for $800,000. How much gain must they recognize on this sale for federal income tax purposes? A) $0 B) $50,000 C) $250,000 D) $300,000

The correct answer is (A). The first $500,000 of gain on the sale of a primary residence is not recognized for federal income tax purposes, therefore, their taxable gain is $0.

The goals of regulatory oversight include all the following EXCEPT A) reducing inequality. B) enhancing transparency. C) increasing integrity. D) maintaining accuracy in financial reporting.

The correct answer is (A). The goals of regulatory oversight are fostering transparency, integrity, and accuracy.

If interest rates increase, then bond prices will _______ while the returns on reinvested coupons will _______. A) decrease, increase B) decrease, decrease C) increase, decrease D) increase, increase

The correct answer is (A). There is an inverse relationship between interest rate risk and reinvestment rate risk. If interest rates increase, bond prices will fall, while reinvest coupons will experience increased returns.

Which of the following is (are) included in a mutual fund's expense ratio? 12b-1 fees Front-end loads A) I only B) II only C) Both I and II D) Neither I nor II

The correct answer is (A). While 12b-1 fees are included in a fund's expense ratio, front-end loads are not.

Colin purchased Apple stock earlier this year. The stock has increased significantly. While Colin likes the company's long-term prospects, he is concerned about a rapid price drop. Which of the following strategies would be protect him from such a price drop? A) Buy Apple puts. B) Sell Apple puts. C) Buy Apple calls. D) Sell Apple calls.

The correct answer is (A). A put option protects against drops in Apple stock price.

According to the Black-Scholes option pricing model, what will happen to the average cost of call options if interest rates increase? A) The average cost will increase. B) The average cost will decrease. C) For interest sensitive stocks, but not for other stocks, the average cost will increase. D) For interest sensitive stocks, but not for other stocks, the average cost will decrease.

The correct answer is (A).According to the Black-Scholes option pricing model, interest rate increases will increase the average cost of all call options. Higher interest rates mean investors make relatively higher returns in stable, less risky investments. Call options allow these investors exposure to price increases in the riskier stock market without committing to large up-front investments. The greater demand for call options, in turn, increases their price.

Gene and Jean are married taxpayers who have a high income and are in the top marginal tax bracket. They own the home they live in. The home has greatly appreciated in value since he bought it and they are concerned about the taxes they will owe if they sell it. Which of the following statements best describe the tax-savings opportunities available to them? A) If they have lived in the home at least 2 of the past 5 years, they will not pay taxes on the first $500,000 of gain. B) They can depreciate the value of the home, saving on taxes today; and, when they sell the home, they can recapture the depreciation at a lower tax rate than the top marginal rate. C) If they perform a Section 1031 exchange, they may be able to defer or avoid taxation on the gain. D) If they gift the home to a relative, that relative will receive a step-up-in-basis; the relative can then sell the home without recognizing any gain.

The correct answer is (A).As a married couple, Gene and Jean can exclude the first $500,000 of gain from their taxable income (assuming they lived in the home at least 2 of the past 5 years). Statements (B) and (C) are incorrect because these rules apply to business property but not personal property. Statement (D) is incorrect because the step-up-in-basis rules apply to inherited property but not gifted property.

Which of the following regarding futures contracts is least accurate? A) Futures contracts are less liquid than forward contracts. B) Futures contracts are marked to market. C) Futures contracts are traded on a regulated exchange. D) Futures contracts allow more delivery options than forward contracts.

The correct answer is (A).Futures contracts are more liquid than forward contracts, because they are traded on organized exchanges and are standardized. Forward contracts are customized to the parties involved in the transaction.

Which type of mutual fund share generally converts to an "A share" after a period of time? A) Class B shares B) Class C shares C) Both B and C shares D) Neither B nor C shares

The correct answer is (A).Most Class B shares have a conversion feature that allows the shares to convert to Class A after a specified time period. Only in certain cases may Class C shares convert to A shares.

Which of the following statements concerning a unit investment trust is correct? A) It has an unmanaged portfolio. B) It usually consists of common stocks. C) It is most appropriate for young people. D) It requires constant management attention

The correct answer is (A).Option (B) is incorrect as UITs generally consist of bonds. Option (C) is incorrect as UITs are better for generating income, not the typical objective of young people. Option (D) is incorrect as the fund is generally passive.

Primary commodities likely include: A) Oil extracted from wells B) Refined gasoline C) Steel produced from iron ore D) "2 by 4s" made in a lumber yard

The correct answer is (A).Primary commodities are those that are considered natural resources that can be mined or extracted and used without much processing needed for consumption. Examples include oil, gold, and wheat. When a commodity must undergo a significant process to be able to be consumed, it is known as a secondary commodity. Examples include refined gas, lumber, and steel.

A writer of a put or call may have tax consequences in all the following situations EXCEPT A) when the option premium is received. B) when the option is exercised. C) when the option is exercised. D) when the option position is closed.

The correct answer is (A).There are no tax consequences when an option is initially written. Tax consequences only arise when the option is exercised, when the option expires, or when the option position is closed with an offsetting trade.

Kay Fields compares the fees and expenses of several equity funds. Fields will likely have the most difficulty in evaluating A) trading costs. B) the 12b-1 fees. C) the back-end loads. D) the front-end loads.

The correct answer is (A).Trading costs are the most difficult for an investor to assess, partly because the other fees and expenses are disclosed to investors in the prospectus, but also because some trading costs are implicit. Examples of implicit trading costs are bid-ask spreads, market-impact costs, and opportunity costs.

James is considering purchasing an 11-year bond that is selling for $1,250. What is the current yield for this bond if it has a 6.5% coupon, paid semiannually? A) 6.5% B) 5.2% C) 4.3% D) 3.7%

The correct answer is (B). Current yield = $65/$1250 = 5.2%

Which of the following statements about the prices in a futures contract is correct? A) As the contract nears its expiration date, the spot price and futures price diverge. B) The futures price is the price of the asset at some time in the future. C) Once a futures contract is purchased, its spot price is fixed. D) A spot price is to a forward contract as a futures price is to a futures contract.

The correct answer is (B). In a futures contract, the futures price is the price of the asset at some time in the future. Statement (A) is incorrect because, as the futures contract nears its expiration date, the spot price and futures price converge. Statement (C) is incorrect because a contract's futures price is fixed once it is purchased. Statement (D) is incorrect because a spot price and futures price apply to both forward and futures contracts.

A real estate investment trust (REIT) manages a portfolio of office buildings that lease space on their ground floor to small shopping centers. In what type of property does this REIT invest? A) Retail B) Mixed-use C) Commercial D) Industrial

The correct answer is (B). Mixed-use property is the name given to buildings that house both office space (commercial) and retail property.

Which of the following would be considered a private-purpose municipal bond? A) A bond issued by a city to help pay for a new police station B) A bond issued by a city to help pay for a new football stadium C) A bond issued by the federal government to help pay for the military D) A bond issued by a corporation to help pay for a new factory

The correct answer is (B). Private-purpose muni bonds are used to finance projects that have some component of private/public use, such as a stadium.

Beverly engaged in several capital transactions this year. She had a short-term capital gain of $400, a short-term capital loss of $600, a long-term capital gain of $800, and a long-term capital loss of $500. How will Beverly report these items on her income tax return? A) Beverly will report a net short-term capital gain of $100. B) Beverly will report a net long-term capital gain of $100. C) Beverly will report a net short-term capital loss of $200 and a net long-term capital gain of $300. D) Beverly will report ordinary income of $100.

The correct answer is (B). Short-term capital losses can be netted against short-term capital gains, and long-term capital losses can be netted against long-term capital gains. Since there is a short-term loss of $200 and a long-term gain of $300, the short- and long-term summary results can be netted together. In this case, since the long-term capital gain was larger on an absolute basis, the character of the resulting net gain is a long-term capital gain.

The creation of which of the following is most often linked to the increased popularity of investment companies? A) Roth IRAs B) 401(k) Plans C) The Investment Company Act of 1940 D) The Investment Advisers Act of 1940

The correct answer is (B). The growth of 401(k) plans in the 1970s and 1980s required individuals to save for their retirement. Investment companies provided an opportunity for investors to save for retirement without dedicating much time and expense to managing their investments.

Investors may benefit from adding alternative investments to their portfolio because their higher expected return tends to increase portfolio returns. their low correlation with traditional asset classes can reduce portfolio risk. A) I only B) II only C) Both I and II D) Neither I nor II

The correct answer is (B). The major benefit of alternative investments is their low correlation with stocks and bonds. This low correlation makes them especially desirable for diversifying a portfolio.

A financial analyst computes the price-to-sales ratio as a comparison to determine which company is most expensive. The analyst is most likely using A) the discounted cash flow technique. B) the relative valuation methodology. C) indexing. D) technical analysis.

The correct answer is (B). The relative valuation method uses price multiples to determine the relative value of each individual stock. The price-to-sales ratio is commonly used for firms that do not have a long history of positive earnings.

When making an asset selection decision, an analyst elects to use the relative valuation approach. Which of the following is a disadvantage of this approach? A) It requires a large number of assumptions to produce an analysis. B) It is difficult to identify acceptable peer groups for comparison. C) It rarely estimates prices that are close to actual market prices. D) It produces analyses that are confusing and counter-intuitive.

The correct answer is (B). With the relative valuation approach, it is often difficult to identify acceptable peer groups for a given company. Analysts must balance firm size, capital structure, and location.

Which of the following statements is/are correct? I. Longer maturity bonds are less volatile than shorter term bonds when interest rates fluctuate. II. Higher coupon bonds are less volatile than lower coupon bonds when interest rates fluctuate. A) I only B) II only C) Both I and II D) Neither I nor II

The correct answer is (B). When interest rates change, bonds with higher coupon payments and shorter maturities are less volatile.

Juan bought XYZ Company stock at $40 per share. Today, the stock sells for $52 per share. Juan likes the long-term prospects for XYZ stock but wants some protection against price decreases. Which of the following orders is the best way for Juan to both participate in future price increases and ensure a minimum profit of $6 per share? A) A limit order to sell at $46 B) A stop-limit order, stop price = $47, limit price = $46 C) A limit order to buy at $40 D) A stop-sell order, stop price = $46

The correct answer is (B).A limit order will not work as it is a sell order at or above $46 that would be executed today. It does not provide the downside protection Juan is looking for. The stop-limit order will place a limit order with a price of $46 once the stock reaches $47. The stop-sell order results in a market order once the stock price reaches $46, which likely will result in the order being filled below $46.

Which of the following is a violation of the wash-sale rule? A) You sell 27 shares of an S&P 500 index fund at a loss. Then 15 days later, you buy 42 shares of a Russell 3000 index fund. B) You have brokerage accounts at two firms. At the first firm, you buy 100 shares of PQR stock. Then 22 days later, you sell 100 shares of PQR stock, bought the prior year, for a loss in your account at the other firm. C) Your spouse buys 200 shares of UK stock. Then 31 days later, you sell 100 shares of the same stock, bought 43 days earlier, at a slight loss. D) You sell 100 shares of UK stock at a slight gain. Then 4 days later, your spouse buys 200 shares of the same stock.

The correct answer is (B).Option (A) is incorrect because the two indices are sufficiently different. Option (C) is incorrect because the purchase is more than 30 days prior to the sale. Option (D) is incorrect because the stock is sold at a gain.

Philip buys undeveloped beachfront property. He is subject to all of the following risks EXCEPT A) Market risk B) Default risk C) Liquidity risk D) Political risk

The correct answer is (B).Philip, as an investor in real estate, is subject to market risk, liquidity risk and political risk. He is not subject to default risk. There is no indication that there is debt involved in this question. In addition, he would not be the one subject to default risk even if debt was involved in the purchase of the real estate.

Which of the following statements regarding the primary market is (are) correct? I. Investment bankers most commonly use best efforts to facilitate primary offerings.II. Initial public offerings are a significant component of the primary market. A) I only B) II only C) Both I and II D) Neither I nor II

The correct answer is (B).Statement I is incorrect because investment bankers most commonly use a firm commitment to facilitate a primary offering.

Which of the following is an index designed to track the performance of large- and mid-cap securities from 21 developed countries? A) Wilshire 21 B) MSCI EAFE C) Barron's 400 D) Euro 21

The correct answer is (B).The MSCI EAFE index is often used as a benchmark for foreign asset managers.

Higgins purchased 2,000 shares of Dunlap, Inc., stock for $28 per share three years ago and paid a $50 brokerage commission on the transaction. This year he sold the shares for $37 per share and paid a $50 brokerage commission on the transaction. What is the amount of his taxable gain or loss? A) $0 B) $17,900 C) $18,000 D) $74,000

The correct answer is (B).The amount realized from the sale is $74,000 − $50 commission = $73,950. His cost basis is $56,000 + $50 commission = $56,050. The taxable gain is $73,950 − $56,050 = $17,900.

Which rebalancing strategy incurs the lowest transaction costs? A) The constant-weighting strategy B) The buy-and-hold strategy C) The tactical allocation strategy D) The dynamic allocation strategy

The correct answer is (B).The buy-and-hold strategy is a strategy that avoids the transaction costs of periodic rebalancing by simply allowing investments to rise or fall in value over time. The primary disadvantage of this strategy is that one's asset allocation will be affected by the past performance of investments held in the portfolio. In order to preserve a desired asset allocation, some other form of rebalancing may be preferable despite the higher transaction costs.

Onion, a large technology company, has stock that pays a $1 dividend semiannually. Its current earnings per share is $10. If the stock is currently trading at $120, what is the dividend yield percentage? A) 0.83% B) 1.67% C) 10.00% D) 20.00%

The correct answer is (B).The dividend yield percentage is equal to the total annual dividends per share divided by the stock price. ($1 × 2) / $120 = 1.67%

Security A has the following returns over 4 years: 4%, 7%, 0%, and -1%. What is the mean return and the standard deviation (sample) for Security A? A) Mean of 2.5% and standard deviation of 3.2% B) Mean of 2.5% and standard deviation of 3.7% C) Mean of 4% and standard deviation of 3.2% D) Mean of 4% and standard deviation of 3.7%

The correct answer is (B).The mean is 2.5 percent and the standard deviation is close to 3.7 percent. You can calculate these using the ∑+ key on a financial calculator.

Three years ago, Ty purchased 1,000 shares of Pope Industries, Inc., for $10 per share. He signed an agreement with the company that allowed the company to use his dividend payments to purchase additional shares for him. Over the last 3 years, Ty received a total of $1,200 in dividend payments, which purchased an additional 100 shares of stock. If Ty sells all of his shares for $24,000, what is his taxable gain? A) $0 B) $12,800 C) $14,000 D) $24,000

The correct answer is (B).Ty's adjusted basis in the shares equals the initial purchase price of the shares, $10,000, plus the dividends of $1,200 that were reinvested, since Ty was required to pay tax on the dividend payments for the year in which they were made. When he sells the shares, he realizes $24,000 less his adjusted basis of $11,200, which equals a taxable gain of $12,800.

When an investor sets a bond portfolio's duration to be equal to the investor's time horizon, they reduce the portfolio exposure to interest rate risk. reinvestment rate risk. A) I only B) II only C) Both I and II D) Neither I nor II

The correct answer is (C). A bond is immunized if its duration is matched to the timing of the investor's cash flow needs. An immunized portfolio has less exposure to both interest rate and reinvestment rate risks.

A mutual fund consisting mostly of large cap domestic stocks is most likely to have a A) high beta relative to the S&P 500. B) low beta relative to the S&P 500. C) high R2 relative to the S&P 500. D) low R2 relative to the S&P 500.

The correct answer is (C). A high R2 relative to the S&P 500 suggests that this portfolio is very similar to the S&P 500. Therefore, this portfolio is most likely to consist of large cap domestic stocks, the same securities that make up the S&P 500.

If an investment pays a fixed 7% rate of return each quarter, what is its annualized return? A) 7% B) 28% C) 31% D) 49%

The correct answer is (C). Annualized return = (1 + 0.07)4 − 1 = 1.074 − 1 = 1.3108 − 1 = 0.3108, or about 31%

A portfolio consists of two uncorrelated assets: A and B. Using the information in the table below, calculate this portfolio's standard deviation. Asset Weight Mean Return Standard Deviation A 80% 6% 8% B 20% 4% 6% A) 1.63% B) 2.76% C) 6.51% D) 7.60%

The correct answer is (C). Portfolio SD = [(0.802 × 8%2) + (0.202 × 6%2)]1/2 Portfolio SD = [(0.64 × 64%) + (0.04 × 36%)] 1/2 Portfolio SD = (40.96% + 1.44%)1/2 Portfolio SD = (42.40%)1/2 = 6.51%

A covered call is the most appropriate strategy for investors looking for A) profit on substantial decreases in a stock's value. B) low purchase prices on a stock they anticipate will increase in value. C) additional income from a stock they already own. D) downside protection on a stock they already own.

The correct answer is (C). Selling covered calls on a stock an investor already owns can help the investor make additional income off the stock.

An investor seeking relatively low default risk investments combined with some type of favorable tax treatment and inflation protection should consider which of the following? A) Revenue municipal bonds B) Series EE savings bonds C) Series I savings bonds D) Series HH savings bonds

The correct answer is (C). Series I savings bonds offer inflation protection and are default-risk-free. Muni bonds and Series EE savings bonds generally do not offer inflation protection. Series HH bonds are no longer sold.

As of 2021, what is the maximum possible federal tax rate applied to capital gains from the sale of common stock? A) 18.8% B) 20.0% C) 23.8% D) 28.0%

The correct answer is (C). The maximum possible capital-gains rate is the base 20% maximum plus the 3.8% net investment income tax for a total of 23.8%.

All the following statements regarding investment strategies for tax-advantaged accounts are correct EXCEPT: A) The general rule is that one should place securities with interest income in qualified accounts and securities with good prospects for capital gains in nonqualified accounts. B) The major tax-advantaged accounts include IRAs and qualified plans through work such as the 401(k). C) Investors should always maximize their tax-advantaged accounts before contributing to taxable brokerage accounts. D) All tax-advantaged accounts allow investors to defer the recognition of taxable income and some even allow for tax-free growth.

The correct answer is (C). There are cases in which investors should prioritize contributing to taxable brokerage accounts (e.g., when creating an emergency fund or a down payment fund). Tax-advantaged accounts generally limit withdrawals, which may subject the investor to additional taxes or penalties if the funds are used for certain nonqualified purposes.

Which of the following bonds are most likely to be called? A) Treasury bonds after interest rates decreased. B) Treasury bonds after interest rates increased. C) Coupon bonds after interest rates decreased. D) Coupon bonds after interest rates increased.

The correct answer is (C). Bonds are called when a company can refinance its debts at a lower interest rate. Treasury bonds are noncallable.

Private equity is regularly characterized by A) the ability to accurately estimate the beta of a target business. B) low funding risk. C) high liquidity risk. D) active secondary markets to trade private equity positions.

The correct answer is (C). It may be difficult for a private equity investor to sell her position. Statement (A) is incorrect because it is very difficult for private equity firms to gain enough access to relevant financial information of target businesses to be able to accurately estimate the target's beta. Statements (B) and (D) are incorrect because PE has significant funding risk at least some of the time, and there is a limited secondary market.

All of the following regarding TIPS are true EXCEPT: A) The interest paid on TIPS changes based on inflation and the changing principal amount of the TIPS. B) TIPS are considered low-risk investments. C) TIPS are most appropriate for young investors with many decades until retirement. D) If deflation occurs over the life of the TIPS, the investor receives the greater of the principal amount or the par value.

The correct answer is (C). TIPS are a low-risk, low-reward investment that only pays well in periods of high inflation. Young people are less exposed to inflation risk because their incomes and higher-risk investments will help keep pace with inflation. Older people into retirement, on the other hand, are very exposed to inflation risk, making TIPS a more appropriate investment for them.

$100 of stock is purchased with a 50% initial margin and a 30% maintenance margin. At what price would a margin call occur? A) $28.58 B) $70.00 C) $71.42 D) $85.00

The correct answer is (C). The formula used to determine the price below when a margin call will occur is: debt / (1 − MMR). For example, if a 50% initial margin rate and a 30% maintenance margin rate apply, a stock that was purchased when the price was $100 would receive a margin call if the price falls below $50 / (1 − 0.30) = $71.42. The price drop is $28.58, or 1 − debt / (1 − MMR).= 1 − (0.50 / 0.70) = 28.58%.

If an investor owns a single share of a stock with a beta of 0.75, what can you conclude about his investment risk relative to the market? A) The investor's total risk is 3/4 of the risk of the market. B) The investor is taking 75 percent more total risk than the risk of the market. C) The investor's systematic risk is ³⁄₄ of the risk of the market. D) The investor's diversifiable risk is 75% more than the total risk of the market.

The correct answer is (C).Beta only measures systematic risk relative to 1, the market's beta. Beta assumes a stock is added to an already diversified portfolio. Because a single stock has both diversifiable as well as systematic risk, an investor who purchases only a single stock may have a total risk greater than the market's risk.

Marleen, who lives in Louisiana, is in the 20 percent federal tax bracket and 5 percent state income tax bracket. Which of the following bonds that she is considering purchasing has the highest after-tax yield? A) A treasury bond paying 4% B) A corporate bond paying 4.5% C) A Louisiana municipal bond paying 3.5% D) A Texas municipal bond paying 3.6%

The correct answer is (C).Corporate bonds are subject to federal and state income tax. Treasury bonds are subject to federal income tax only. Municipal bonds are not subject to federal income tax, but they are subject to state income tax if they are not issued by the taxpayer's state of residence. The La. muni has the highest after-tax return.Treasury = 4%*(1-.2)=3.200% Corporate = 4.5*(1-.25)=3.375% La. Bond = 3.500% Tex. Bond = 3.42%

Cashflow, Inc. expects to generate $200 million in operating cash flows during the next year. It estimates its long-term dividend growth rate to be 3 percent, and it has 100 million shares outstanding. What is the intrinsic value of Cashflow, Inc., if your required rate of return is 9 percent? A) $16.67 B) $22.22 C) $33.33 D) $66.67

The correct answer is (C).Intrinsic value = Total market value ÷ Outstanding SharesTotal market value = Operating Cash Flow ÷ (Required Rate of Return - Dividend Growth Rate) Total market value = $200,000,000 ÷ (0.09 − 0.03) = $200,000,000 ÷ 0.06 = $3,333,333,333Intrinsic value = $3,333,333,333 ÷ 100,000,000 = $33.33

Sam's retirement fund is expected to earn a nominal rate of 7 percent, and the inflation rate is estimated at 3 percent. What is Sam's real rate of return? A) 1.43% B) 2.33% C) 3.88% D) 4.00%

The correct answer is (C).Real return = (1.07 ÷ 1.03) − 1 = 3.8835%

Which of the following techniques or strategies would take advantage of a perceived undervaluation in the energy sector of the economy? A) Dollar-cost averaging into a portfolio B) Strategic asset allocation rebalancing C) Tactical asset allocation D) Index funds

The correct answer is (C).Tactical asset allocation is in response to short-term market conditions. Dollar-cost averaging is a method of investing over time. Strategic asset allocation is a long-term strategy, not one that takes advantage of sector imbalances.

Mutual fund 12b-1 fees cover which of the following? A) Portfolio management fees B) Administrative costs C) Advertising and marketing fees D) Organizational costs

The correct answer is (C).The 12b-1 fee structure allows for up to 1% allowance for distribution costs, including advertising and promotion.

Which of the following can be greatly reduced by diversification? A) Systematic risk B) Market risk C) Unsystematic risk D) Systematic and unsystematic risk

The correct answer is (C).Unsystematic risk is reduced by diversification.

All the following statements regarding collateralized mortgage obligations (CMOs) are true EXCEPT: A) CMOs are subject to prepayment risk. B) CMOs allow investors to select the tranche that best meets their risk-return needs. C) CMOs are secured bonds. D) CMOs are issued by the U.S. government.

The correct answer is (D). CMOs are bonds created by private investment firms and are secured by a pool of mortgages. Since they are mortgage-backed securities, they are subject to prepayment risk. The advantage of a CMO is the ability to select the tranche that best meets the investor's risk-return needs.

An investor buys $50,000 worth of shares in a mutual fund. In the first year, the fund generates $1,000 of income. In the second year, it generates $2,000 of income. At the end of the 3rd year, the fund generates $500 of income and the shares are worth $55,000. What was this investor's holding period return for the past 3 years? A) 5.51% B) 6.36% C) 10.00% D) 17.00%

The correct answer is (D). HPR = [(Ending balance − Beginning balance) + income)]/Beginning balance HPR = [(55,000 − 50,000) + (1,000 + 2,000 + 500)]/50,000 HPR = [(5,000 + 3,500)/50,000 = 0.17, or 17%

Ingrid owns a bond that pays a semi-annual coupon. The last coupon was $20 and was paid 3 months ago. The current list price for the bond is $1,100. If Ingrid sells the bond, how much will she receive (ignoring any trading fees and taxes)? A) $1,080 B) $1,090 C) $1,100 D) $1,110

The correct answer is (D). Ingrid will receive the list price plus half of the accrued interest.

All the following statements about alternative investments are correct EXCEPT: A) They include commodities and collectibles. B) They are purchased for their low correlation to equities and bonds. C) They are actively managed and are associated with high fees. D) They generally provide high liquidity and appreciation.

The correct answer is (D). Statement (D) is incorrect because alternative investments generally have low liquidity.

All the following statements about all investment companies are correct EXCEPT: A) They provide investors with easier access to markets. B) They provide a diversified portfolio created more efficiently than if constructed from individual securities. C) They provide individual investors with investment opportunities otherwise available only to institutional investors. D) They provide a personal relationship with an investment manager.

The correct answer is (D). Statement (D) is incorrect because investors will rarely have a personal relationship with the investment manager of an investment company; this is particularly true for investors in ETFs and closed-ended funds, which trade on the secondary market.

Which of the following statements regarding mutual funds is (are) true? In a typical year, mutual fund managers make substantial shifts to their funds' investments. Mutual funds regularly issue new shares on the secondary market in anticipation of demand. A) I only B) II only C) Both I and II D) Neither I nor II

The correct answer is (D). Statement I is incorrect because mutual fund managers rarely make substantial shifts to their funds' investments. Statement II is incorrect because investors directly buy shares from the investment manager; shares are never available in the secondary market.

What is the primary risk to investors when hedging with futures contracts? A) The spot price may increase. B) Futures prices may increase. C) Futures prices may decrease. D) The investor has insufficient funds to meet margin requirements.

The correct answer is (D). The biggest risk when hedging with a futures contract is the required margin requirement funds necessary to maintain the hedged position. The purpose of the hedge is to mitigate the risk of price fluctuations.

Christie bought XYZ stock for $7 per share 3 months ago. She sold it today for $8 per share. What is her annualized return? A) 28% B) 31% C) 49% D) 71%

The correct answer is (D). This investor's return over the past three months was 14.29%, or ($8 − $7)/$7 = $1/$7 = 0.1429. Annualized return = (1 + 0.1429)4 − 1 = (1.1429)4 − 1 = 1.7060 − 1 = 0.7060, or about 71%

All of the following are likely to be classified as investment advisers by the U.S. Securities and Exchange Commission (SEC) EXCEPT A) a financial analyst who issues a new report on the performance of value stocks directly to a client. B) an analyst who recommends a triple-A-rated bond to a client. C) a money manager who makes asset allocation assignments for institutional investors. D) a financial firm that acts as a dealer in investment grade bonds.

The correct answer is (D). Broker-dealers are explicitly excluded from being classified as advisers under the legal framework of the Investment Advisers Act. Individuals or firms that issue reports or offer investment advice, including asset-allocation decisions, are considered to fit the definition of an investment adviser.

A contract that is an agreement for immediate exchange of funds for assets is called a A) futures contract. B) forward contract. C) call option contract. D) spot contract.

The correct answer is (D).A spot contract is a contract that calls for immediate delivery of goods. The spot price or rate is the current price or rate.

The return from the CAPM is which of the following? A) The risk-free return B) The actual return C) The risk-premium D) The expected return

The correct answer is (D).CAPM = Rf + β(Rm - Rf ) = the expected return based on beta, the market premium, and the risk-free rate of return

All of the following positions are examples of the types of holdings found in a hedge fund manager's portfolio EXCEPT A) the long-only position in a gasoline futures contract. B) a put option written on a barrel of oil. C) long and short positions in the equity of two industry leaders. D) the long position only in a highly rated municipal bond.

The correct answer is (D).Hedge funds rarely invest in investment grade municipal bonds, because the risk is generally very low and the return is not sufficient to warrant interest by the hedge fund manager. Hedge funds often invest in options and futures contracts and also try to benefit when one industry leader outperforms its main rival.

Which is the least likely motivation for investing in mutual funds? A) To seek diversification benefits B) To obtain professional management of investments C) To invest conveniently D) To obtain returns that normally exceed market returns

The correct answer is (D).Mutual funds offer diversification, convenience, and professional management, but they frequently perform well below the market return.

Maggie considers four potential securities with identical expected returns but different correlations with her existing portfolio as part of her asset-selection decision. Which security would provide Maggie with the greatest diversification benefit? A) Security 1 with a correlation coefficient of +1.0 B) Security 2 with a correlation coefficient of +0.32 C) Security 3 with a correlation coefficient of 0.00 D) Security 4 with a correlation coefficient of -0.38

The correct answer is (D).Since the returns of the four potential securities are the same, the primary consideration is the risk element. The best security is the one with the lowest correlation, which is Security 4.

Cody purchased 400 shares of NAY stock six years ago when it was trading at $65 per share. Unfortunately, NAY has been steadily declining. Cody sold his shares this year for $18 per share. This year Cody also sold 800 shares of a mutual fund that he purchased 6 months ago. His average cost per share was $15 and he sold the shares for $32. Assuming Cody had no other capital transactions this year, what is Cody's tax consequence from these transactions? A) $5,200 long-term loss this year B) $18,800 long-term loss, $13,600 short-term gain taxed at ordinary income rates this year C) $3,000 ordinary loss this year with a $15,800 ordinary loss carried forward up to 5 years D) $3,000 ordinary loss this year with a $2,200 loss carried forward indefinitely

The correct answer is (D).The NAY stock sale resulted in a long-term loss of $18,800 ($47 loss per share × 400 shares). The mutual fund sale resulted in a short-term gain of $13,600 ($17 gain × 800 shares). The entire $13,600 gain is offset by the loss, leaving a net long-term loss of $5,200. Cody can take $3,000 of the loss against ordinary income this year, and the remaining loss is carried forward indefinitely (until death).

All of the following is (are) among the responsibilities of FINRA EXCEPT A) educating investors. B) fostering market transparency. C) writing and enforcing rules governing the activities of all registered broker-dealer firms and registered brokers in the U.S. D) writing and enforcing rules governing the activities of all registered investment advisors.

The correct answer is (D).The SEC and state securities regulators are responsible for regulation of registered investment advisors.

A real estate investor bought a property for $150,000 and rented it out over 3 years before selling it for $160,000. His income and expenses over this period are shown in the table below. Based on this information, what was his dollar-weighted return? YearIncomeExpenses1$7,000-$10,0002$9,000-$10,0003$10,000-$1,000 A) 1.39% B) 2.40% C) 3.18% D) 3.33%

Then, to solve this problem using an HP10bII+, use the following keystrokes: 150,000 [+/-] [CFj] 3,000 [+/-] [CFj] 1,000 [+/-] [CFj] 169,000 [CFj] [SHIFT] [IRR/YR] The correct answer is 3.1809, or about 3.18%.


Kaugnay na mga set ng pag-aaral

Campbell AP Biology Mastering Biology Chapter 23 Work

View Set

Ch 15: Animation: Autonomic Nervous System

View Set

THEA 2013 Quiz Reviews (Modules 1-7)

View Set

History & Geography 808: Twentieth Century World Power - Cost of Conflict

View Set

Chapter 1 Understanding Evolving Economics

View Set

Capital Structure Theory CH 17+18

View Set